1
$\begingroup$

If R is a commutative ring with unity and not an integral domain and F is a free R-module with rank k,is there a linear independent set with cardinality > k? I prooved that this is not true if R is an integral domain and is true if R is not a commutative ring but i can't see the answer to my question.

$\endgroup$
3

1 Answer 1

1
$\begingroup$

Thank you all for your help.The best 2 answers i found are the one of Robin Chapman in this site and the similar one of Papaioannou in the Atiyah-MacDonald solution manual(http://dangtuanhiep.files.wordpress.com/2008/09/papaioannoua_solutions_to_atiyah.pdf)

$\endgroup$

Your Answer

By clicking “Post Your Answer”, you agree to our terms of service and acknowledge you have read our privacy policy.

Not the answer you're looking for? Browse other questions tagged or ask your own question.